Buscar

Curso de Análise Vol 1 Exercícios Resolvidos ANSELMO

Prévia do material em texto

Um Curso de Ana´lise Vol 1 - Exerc´ıcios Resolvidos
Anselmo B. Raposo Ju´nior
Elivaldo Rodrigues Macedo
Jose´ Antoˆnio Pires Ferreira Mara˜o
Suma´rio
Suma´rio 2
1 Sequeˆncias e Se´ries de Nu´meros Reais 1
2 Topologia da Reta 33
2
Cap´ıtulo 1
Sequeˆncias e Se´ries de Nu´meros Reais
1. Se lim xn = a enta˜o lim |xn| = |a|. Deˆ um contra-exemplo mostrando que a rec´ıproca e´
falsa, salvo quando a = 0.
SOLUC¸A˜O: Uma vez que lim xn = a, temos que dado ε > 0, existe n0 ∈ N tal que
|xn − a| < ε, (1.1)
sempre que n > n0. Mas,
||xn| − |a|| ≤ |xn − a| (1.2)
para todo n ∈ N e, consequentemente, de (1.1) e (1.2) segue que
||xn| − |a|| < ε,
sempre que n > n0, ou seja, lim |xn| = |a|.
Considere agora a sequeˆncia xn = (−1)n. Temos que lim |xn| = 1, mas a sequeˆncia (xn)
e´ divergente pois possui duas subsequeˆncias que convergem para valores distintos, a saber,
x2n−1 = −1 e x2n = 1. �
2. Seja lim xn = 0. Para cada n ponha yn = min {|x1| , |x2| , . . . , |xn|}. Prove que yn → 0.
SOLUC¸A˜O: Suponha que yn na˜o converge para 0. Deste modo, existe ε > 0 tal que, para
todo n ∈ N,
|xn| > ε,
contradizendo o fato de xn → 0. Logo yn → 0. �
3. Se lim x2n = a e lim x2n−1 = a, prove que lim xn = a.
SOLUC¸A˜O: Uma vez que lim x2k = a e lim x2k−1 = a, dado ε > 0, existe k0 ∈ N tal que
|x2k − a| < ε e |x2k−1 − a| < ε, (1.3)
sempre que k > k0. Fac¸a n0 = 2k0 e seja n > n0. Se n ∈ N e´ par, existe k ∈ N tal que
n = 2k. Uma vez que n > n0, temos que 2k > 2k0 e, consequentemente, k > k0. Portanto,
de (1.3) segue que
|xn − a| = |x2k − a| < ε. (1.4)
1
Por outro lado, se n e´ ı´mpar, enta˜o existe k ∈ N tal que n = 2k − 1. Como n > n0, temos
que 2k > 2k − 1 > 2k0, isto e´, k > k0. Logo, de (1.3) segue que
|xn − a| = |x2k−1 − a| < ε. (1.5)
De (1.4) e (1.5) segue que para todo n ∈ N tal que n > n0 temos que
|xn − a| < ε,
ou seja, lim xn = a. �
4. Se N = N1 ∪ N2 ∪ · · · ∪ Nk e lim
n∈N1
xn = lim
n∈N2
xn = · · · = lim
n∈Nk
xn = a, enta˜o, lim xn = a.
SOLUC¸A˜O: Para cada i = 1, . . . , k temos que lim
n∈Ni
xn = a, isto e´, dado ε > 0, existe n0i ∈ Ni
tal que
|xn − a| < ε
sempre que n ∈ Ni e n > n0i. Fac¸a n0 = max {n01, . . . , n0k}. Deste modo, para todo
n > n0 ∈ N, existe i ∈ {1, . . . k} tal que n ∈ Ni e, consequentemente,
|xn − a| < ε,
isto e´, lim xn = a. �
5. Deˆ um exemplo de uma sequeˆncia (xn) e uma decomposic¸a˜o N = N1 ∪ · · · ∪ Nk ∪ · · ·
de N como reunia˜o de uma infinidade de subconjuntos infinitos tais que, para todo k, a
subsequeˆncia (xn)n∈Nk tenha limite a, mas na˜o se tem lim xn = a.
[Sugesta˜o: Para cada k ∈ N seja Nk o conjunto dos nu´meros naturais da forma n = 2k−1 ·m,
onde m e´ ı´mpar. Dado n ∈ Nk, ponha xn = 1 se n for o menor elemento de Nk e xn = 1
n
,
nos demais casos.]
SOLUC¸A˜O: Para cada k ∈ N, seja Nk =
{
n ∈ N;n = 2k−1 ·m, m ı´mpar}. Se n ∈ N e´ ı´mpar,
enta˜o n ∈ N1. Se n e´ par, enta˜o seja k − 1 o maior nu´mero natural tal que 2k−1 divide
n. Neste caso, existe m ∈ N tal que n = 2k−1 ·m. Note que m deve ser ı´mpar, pois, caso
contra´rio, n seria divis´ıvel por 2k, contrariando a maximalidade de k−1. Note ainda que, da
unicidade da fatorac¸a˜o prima dos nu´meros naturais, segue que os subconjuntos Nk sa˜o dois
a dois disjuntos. Logo N =
∞⋃
k=1
Nk. Dado n ∈ Nk, ponha xn = 1 se n for o menor elemento
de Nk e xn =
1
n
nos demais casos. Enta˜o lim
n∈Nk
xn = 0. Mas (xn)n∈N na˜o converge, pois se
tomamos
N′ = {n ∈ N; n e´ o menor elemento de Nk, para algum k ∈ N} ,
enta˜o lim
n∈N′
xn = 1, isto e´, existem subsequeˆncias de (xn)n∈N que convergem para valores
distintos. �
6. Se lim xn = a e lim (xn − yn) = 0 enta˜o lim yn e´ igual a a.
2
SOLUC¸A˜O: Uma vez que lim xn = a e lim (xn − yn) = 0 segue que, dado ε > 0, existe n0 ∈ N
tal que
|xn − a| < ε
2
e |xn − yn| < ε
2
,
sempre que n > n0. Deste modo,
|yn − a| = |yn − xn + xn − a| ≤ |xn − yn|+ |xn − a| < ε
2
+
ε
2
= ε,
sempre que n > n0, isto e´, lim yn = a. �
7. Seja a 6= 0. Se lim yn
a
= 1 enta˜o lim yn e´ igual a a.
SOLUC¸A˜O: Uma vez que lim
yn
a
= 1, temos que, dado ε > 0, existe n0 ∈ N tal que
∣∣∣yn
a
− 1
∣∣∣ < ε|a| ,
sempre que n > n0. Mas ∣∣∣yn
a
− 1
∣∣∣ =
∣∣∣∣1a (yn − a)
∣∣∣∣ = 1|a| |yn − a|
e, consequentemente, sempre que n > n0,
1
|a| |yn − a| <
ε
|a| ,
ou seja,
|yn − a| < ε
e lim yn = a. �
8. Seja b 6= 0. se lim xn = a e lim xn
yn
= b enta˜o lim yn =
a
b
.
SOLUC¸A˜O: Temos que
lim yn = lim
xn(
xn
yn
) = lim xn
lim
xn
yn
=
a
b
.
�
9. Se lim xn = a 6= 0 e lim xnyn = b enta˜o lim yn = b
a
.
SOLUC¸A˜O: Temos que
lim yn = lim
xnyn
xn
=
lim xnyn
lim xn
=
b
a
.
�
3
10. Sejam k ∈ N e a > 0. Se a ≤ xn ≤ nk para todo n, enta˜o lim n√xn = 1.
SOLUC¸A˜O: Com efeito, uma vez que lim n
√
a = 1 = lim
n
√
nk e a ≤ xn ≤ nk para todo n,
segue do Teorema do Confronto que lim n
√
xn = 1. �
11. Use a desigualdade entre as me´dias aritme´tica e geome´trica dos n + 1 nu´meros 1 −
1
n
, . . . , 1− 1
n
, 1 e prove que a sequeˆncia
(
1− 1
n
)n
e´ crescente. Conclua que
(
1− 1
n
)n
≥ 1
4
para todo n > 1.
SOLUC¸A˜O: Da desigualdade entre as me´dias aritme´tica e geome´trica segue que(
1− 1
n
)
+ · · ·+
(
1− 1
n
)
+ 1
n+ 1
=
n
(
1− 1
n
)
+ 1
n− 1
=
n− 1 + 1
n+ 1
= 1− 1
n+ 1
> n+1
√(
1− 1
n
)
· · ·
(
1− 1
n
)
· 1
= n+1
√(
1− 1
n
)n
.
Asim,
n+1
√
xn+1 =
n+1
√(
1− 1
n+ 1
)n+1
= 1− 1
n+ 1
> n+1
√(
1− 1
n
)n
= n+1
√
xn,
ou seja, xn+1 > xn para todo n > 1. Para n = 2, temos que
x2 =
(
1− 1
2
)2
=
1
4
.
Uma vez que a sequeˆncia (xn)n∈N e´ crescente, temos, para todo n > 1, que xn ≥ x2, isto e´,(
1− 1
n
)n
≥ 1
4
.
�
11a. Sejam xn =
(
1 +
1
n
)n
e yn =
(
1− 1
n+ 1
)n+1
. Mostre que lim xnyn = 1 e deduza da´ı
que lim
(
1− 1
n
)n
= e−1.
4
SOLUC¸A˜O: Temos que
xnyn =
(
1 +
1
n
)n
·
(
1− 1
n+ 1
)n+1
=
(
n+ 1
n
)n
·
(
n
n+ 1
)n+1
=
n
n+ 1
=
(n+ 1)− 1
n+ 1
=
(
1− 1
n
)
.
Logo,
lim xnyn = lim
(
1− 1
n
)
= 1.
Sabemos que e = lim
(
1 +
1
n
)n
= lim xn. Deste modo,
lim
(
1− 1
n+ 1
)n+1
= lim yn = lim
xnyn
xn
=
lim xnyn
lim xn
=
1
e
.
Uma vez que o limite de uma sequeˆncia na˜o se altera se nela acrescentamos ou omitimos
uma quantidade finita de termos segue que
lim
(
1− 1
n
)n
= lim
(
1− 1
n+ 1
)n+1
=
1
e
.
�
12. Fazendo y = x
1
k e b = a
1
k na identidade yk − bk = (y − b)
k∑
i=0
yibk−i−1 obtenha x − a =(
x
1
k − a 1k
) k∑
i=0
x
i
k · a k−i−1k e use isso para provar que se lim xn = a > 0, enta˜o lim k√xn = k
√
a.
Conclua da´ı que lim (xn)
r = ar para todo racional r.
SOLUC¸A˜O: Fazendo y = x
1
k e b = a
1
k na identidade indicada temos que
x− a =
(
x
1
k − a 1k
) k∑
i=0
(
x
1
k
)i (
a
1
k
)k−i−1
=
(
x
1
k − a 1k
) k∑
i=0
x
i
k · a k−i−1k .
Sabemos que xn → a⇔ xn − a→ 0. Seja 0 < c < a. Do Teorema da Conservac¸a˜o do Sinal
segue que existe n0 ∈ N tal que, para todo n > n0, tem-se xn > c. Deste modo, para n > n0,
temos que
xn − a =
[
(xn)
1
k − a 1k
] k∑
i=0
(xn)
i
k · a k−i−1k ≥
[
(xn)
1
k − a 1k
] k∑
i=0
c
i
k · a k−i−1k ≥ 0.
Fazendo K =
k∑
i=0
c
i
k · a k−i−1k , temos que
0 ≤ (xn)
1
k − a 1k ≤ xn − a
K
, ∀ n > n0.
5
Uma vez que
lim
xn − a
K
=
1
K
lim (xn − a) = 0,
segue do Teorema do Confronto que lim
[
(xn)
1
k − a 1k
]
= 0, isto e´, lim k
√
xn = k
√
a. Seja agora
r ∈ Q. Destemodo, existem inteiros j e k, com k 6= 0, tais que r = j
k
. Se j = 0, enta˜o,
obviamente, lim (xn)
r = ar. Se j > 0, enta˜o
lim (xn)
r = lim (xn)
j
k = lim (xn)
1
k · · · (xn)
1
k
=
[
lim (xn)
j
k
]
· · ·
[
lim (xn)
j
k
]
= a
1
k · · · a 1k = a jk = ar.
Segue das propriedades operato´ras de limite de sequeˆncias que se lim xn = a enta˜o lim
1
xn
=
1
a
. Portanto o caso j < 0 reduz-se ao anterior se fizermos yn =
1
xn
. �
13. Prove que, para todo r ∈ Q, tem-se lim
n→∞
(
1 +
r
n
)
= er.
[Sugesta˜o: Pelo Exerc´ıcio 11, basta considerar o caso em que r =
p
q
> 0. Examine a
subsequeˆncia onde n = p ·m. Para estes valores de n, tem-se
(
1 +
r
n
)n
=
[(
1 +
1
qm
)qm] p
q
.
Use o Exerc´ıcio 12]
SOLUC¸A˜O: O caso r = 0 e´ o´bvio. Do Exerc´ıcio 11 segue que e´ suficiente considerarmos o
caso em r =
p
q
> 0. Consideremos a subsequeˆncia (xpm)m∈N. Neste caso temos
xpm =
(
1 +
r
pm
)pm
=
(
1 +
p
q
1
pm
)pm
=
(
1 +
1
qm
) pqm
q
=
[(
1 +
1
qm
)qm] p
q
=
[(
1 +
1
qm
)qm]r
.
Como lim
m→∞
(
1 +
1
qm
)qm
= e segue, do Exerc´ıcio 12, que
lim
m→∞
xpm = lim
m→∞
[(
1 +
1
qm
)qm]r
= er.
Para todo n ∈ N, temos que �
6
(
1 +
r
n
)n
=
n∑
k=0
(
n
k
)( r
n
)k
= 1 +
n∑
k=1
(
n
k
)( r
n
)k
= 1 +
n∑
k=1
n!
k! (n− k)!
( r
n
)k
= 1 +
n!
1! (n− 1)!
r
n
+
n!
2! (n− 2)!
( r
n
)2
+ · · ·+ n!
n! (n− n)!
( r
n
)n
= 1 + r +
n (n− 1)
2!
( r
n
)2
+ · · ·+ n (n− 1) · · · 2 · 1
n!
( r
n
)n
= 1 + r +
r2
2!
(
1− 1
n
)
+ · · ·+ r
n
n!
(
1− 1
n
)
· · ·
(
1− n− 1
n
)
≤ 1 + r + r
2
2!
+ · · ·+ r
n
n!
<
∞∑
n=1
rn
n!
.
O teste da raza˜o nos da´ que
lim
∣∣∣∣rn+1/ (n+ 1)!rn/n!
∣∣∣∣ = lim rn+1(n+ 1)! n!rn = lim rn+ 1 = 0
e, consequentemente, a se´rie
∞∑
n=1
rn
n!
e´ absolutamente convergente e a sequeˆncia xn =
(
1 +
r
n
)n
e´ limitada. Observe que
xn = 1 + r +
r2
2!
(
1− 1
n
)
+ · · ·+ r
n
n!
(
1− 1
n
)
· · ·
(
1− n− 1
n
)
< 1 + r +
r2
2!
(
1− 1
n+ 1
)
+ · · ·+ r
n
n!
(
1− 1
n+ 1
)
· · ·
(
1− n− 1
n+ 1
)
< 1 + r +
r2
2!
(
1− 1
n+ 1
)
+ · · ·+ r
n
(n+ 1)!
(
1− 1
n+ 1
)
· · ·
(
1− n
n+ 1
)
= xn+1
Logo (xn) e´ uma sequeˆncia mono´tona. Deste modo, (xn) e´ uma sequeˆncia convergente e,
portanto,
lim
n→∞
xn = lim
m→∞
xpm = e
r.
14. Seja a ≥ 0, b ≥ 0. Prove que lim
n→∞
n
√
an + bn = max {a, b}.
SOLUC¸A˜O: Sem perda de geberalidade, suponha que seja b = max {a, b}. Deste modo, para
todo n ∈ N, temos que
n
√
an + bn ≤ n
√
bn + bn =
n
√
2bn = b
n
√
2
e
n
√
an + bn ≥ n
√
bn = b.
Mas, lim b = b = lim b n
√
2 e, portanto, segue do Teorema do Confronto que lim n
√
an + bn =
b = max {a, b}. �
7
15. Dada uma sequeˆncia (xn), um termo xp chama-se um “termo destacado” quando xp ≥ xn
para todo n > p. Seja P = {p ∈ N; xp e´ destacado}. Se P = {p1 < p2 < · · · } for infinito,
(xp)p∈P e´ uma subsequeˆncia na˜o-crescente de (xn). Se P for finito (em particula, vazio),
mostre que existe uma subsequeˆncia crescente de (xn). Conclua que toda sequeˆncia possui
uma subsequeˆncia mono´tona.
SOLUC¸A˜O: Seja (xn)n∈N uma sequeˆncia nume´rica e seja P = {p ∈ N; xp e´ destacado}. Se
P e´ um conjunto finito enta˜o existe p0 ∈ P tal que p0 e´ o maior elemento de P . Seja
n1 ∈ N tal que n1 > p0. Como n1 /∈ P , xn1 na˜o e´ um termo destacado e, portanto, existe
n2 ∈ N tal que n2 > n1 e xn2 > xn1 . Do mesmo modo, como n2 /∈ P , xn2 na˜o e´ um termo
destacado e, portanto, existe n3 ∈ N tal que n3 > n2 e xn3 > xn2 . Procedendo deste modo,
obtemos um conjunto infinito de ı´ndices N′ = {nk; k ∈ N} tal que xnk+1 > xnk , para todo
k ∈ N. Logo, (xn)n∈N′ e´ uma subsequeˆncia crescente de (xn)n∈N. Se P for infinito, (xp)p∈P
e´ uma subsequeˆncia na˜o-crescente de (xn). Em todo caso (xn)n∈N possui uma subsequeˆncia
mono´tona, como quer´ıamos demonstrar. �
16. Seja (xn) uma sequeˆncia limitada. Se lim an = a e cada an e´ um valor de adereˆncia de
(xn), enta˜o a e´ um valor de adereˆncia de (xn).
SOLUC¸A˜O: Por hipo´tese, lim
n→∞
an = a e, para todo n ∈ N, existe uma subsequeˆncia (xn,k)k∈N
de (xn)n∈N tal que lim
k→∞
xn,k = an. Assim, dado m ∈ N, existem nm, km ∈ N tais que
|anm − a| <
1
2m
e |xnm,km − anm | <
1
2m
.
Mas,
|xnm,km − a| = |xnm,km − anm + anm − a|
≤ |anm − a|+ |xnm,km − anm |
<
1
2m
+
1
2m
=
1
m
.
Logo a subsequeˆncia (xnm,km)m∈N converge para a e, consequentemente, a e´ um valor de
adereˆncia de (xn). �
17. Sejam (xn) e (yn) sequeˆncias limitadas. Ponhamos a = lim inf xn, A = lim sup xn,
b = lim inf yn e B = lim sup yn. Prove que
a) lim sup (xn + yn) ≤ A+ B, lim inf (xn + yn) ≥ a+ b;
b) lim sup (−xn) = −a, lim inf (−xn) = −A;
c) lim sup (xn · yn) ≤ AB, lim inf (xn · yn) ≥ ab;
valendo as duas u´ltimas desigualdades sob a hipo´tese de xn ≥ 0 e yn ≥ 0. Deˆ exemplos em
que se tenham desigualdades estritas nas relac¸o˜es acima.
8
SOLUC¸A˜O: Para todo n ∈ N, sejam Xn = {xn, xn+1, . . .}, Yn = {yn, yn+1, . . .} e Zn =
{xn + yn, xn+1 + yn+1, . . .}. Sejam ainda an = infXn, An = supXn, bn = inf Yn, Bn =
supYn, cn = inf Zn e Cn = supZn.
(a) Fixe n ∈ N. Se m ≥ n, xm ∈ Xn, ym ∈ Yn e, consequentemente,
xm ≤ An, an ≤ xm, ym ≤ Bn, bn ≤ ym,
para todo m ≥ n. Deste modo,
xm + ym ≤ An + Bn e an + bn ≤ xm + ym,
para todo m ≥ n e, assim, An +Bn e´ uma cota superior de Zn e an + bn e´ uma cota inferior
de Zn. Da´ı
Cn ≤ An + Bn e an + bn ≤ cn.
Observe que as desigualdades acima sa˜o va´lidas para todo n ∈ N. Desta forma
lim sup (xn + yn) = limCn ≤ lim (An + Bn) = A+ B
e
lim inf (xn + yn) = lim cn ≥ lim (an + bn) = a+ b.
(b) Seja −Xn = {−xn,−xn+1, . . .}. Para todo m ≥ n vale xm ≥ an e, consequentemente,
−xm ≤ −an, para todo m ≥ n. Logo −an e´ uma cota superior de −Xn. Afirmamos que
deve ser −an = sup (−Xn). Com efeito, se −an na˜o fosse o supremo de −Xn deveria existir
ε > 0 tal que −xm ≤ −an − ε, para todo m ≥ n. Assim, xm ≥ an + ε, para todo m ≥ n,
ou seja, an + ε e´ uma cota inferior de Xn maior do que an, contrariando o fato de an ser o
ı´nfimo de Xn. Portanto,
lim sup (−xn) = lim (−an) = −a.
De modo ana´logo segue que lim inf (−xn) = −A.
(c) Uma vez que xn ≥ 0 e yn ≥ 0 para todo n ∈ N, temos que an ≥ 0 e bn ≥ 0 para
todo n ∈ N. Fixe n ∈ N. Para todo m ≥ n, temos que 0 ≤ an ≤ xm e 0 ≤ bn ≤ xm.
Logo, xmym ≥ anbn para todo m ≥ n e, portanto, anyn e´ uma cota inferior de XnYn =
{xnyn, xn+1yn+1, . . .}. Fazendo dn = infXnYn segue que dn ≥ anbn. Assim,
lim inf xnyn = lim dn ≥ lim anbn = ab.
A outra desigualdade e´ obtida de modo ana´logo. �
18. Para cada n ∈ N, seja 0 ≤ tn ≤ 1. Se lim xn = lim yn = a, prove que
lim [tnxn + (1− tn) yn] = a.
SOLUC¸A˜O: Uma vez que lim xn = lim yn = a, dado ε > 0, existe n0 ∈ N tal que
|xn − a| < ε
3
e |yn − a| < ε
3
,
9
sempre que n > n0. Deste modo,
|tnxn + (1− tn) yn − a| = |tn (xn − yn) + yn − a|
≤ tn |xn − yn|+ |yn − a|
≤ |xn − yn|+ |yn − a|
= |xn − a+ a− yn|+ |yn − a|
≤ |xn − a|+ 2 |yn − a|
<
ε
3
+
2ε
3
= ε,
sempre que n > n0, ou seja, lim [tnxn + (1− tn) yn] = a. �
19. Diz-se que uma sequeˆncia (xn) tem variac¸a˜o limitada quando a sequeˆncia (vn) dada por
vn =
n∑
i=1
|xi+1 − xi| e´ limitada. Prove que, nesse caso, (vn) converge. Prove tambe´m:
a) Se (xn) tem variac¸a˜o limitada, enta˜o existe lim xn;
b) Se |xn+2 − xn+1| ≤ c |xn+1 − xn| para todo n ∈ N com 0 ≤ c < 1, enta˜o (xn) tem
variac¸a˜o limitada;
c) (xn) tem variac¸a˜o limitadase, e somente se, xn = yn−zn onde (yn) e (zn) sa˜o sequeˆncias
na˜o-decrescentes limitadas;
d) Deˆ exemplo de uma sequeˆncia convergente que na˜o seja de variac¸a˜o limitada.
SOLUC¸A˜O: Observe que
vn+1 =
n+1∑
i=1
|xi+1 − xi| = |xn+2 − xn+1|+
n∑
i=1
|xi+1 − xi| ≥
n∑
i=1
|xi+1 − xi| = vn.
Logo, a sequeˆncia (vn) e´ mono´tona na˜o-decrescente e, portanto, se for limitada, sera´ tambe´m
convergente.
(a) Suponha que (xn) tem variac¸a˜o limitada. Enta˜o, como provado acima, a sequeˆncia
(vn) e´ convergente e, consequentemente, (vn) e´ uma sequeˆncia de Cauchy. Deste modo, dado
ε > 0, existe n0 ∈ N tal que, sempre que m > n− 1 > n0,
|vm − vn−1| =
∣∣∣∣ m∑
i=1
|xi+1 − xi| −
n−1∑
i=1
|xi+1 − xi|
∣∣∣∣ = m∑
i=n
|xi+1 − xi| < ε.
Assim,
|xm − xn| =
∣∣∣∣∣
m∑
i=n
(xi+1 − xi)
∣∣∣∣∣ ≤ ε
sempre que m,n ≥ n0. Logo, a sequeˆncia (xn) e´ uma sequeˆncia de Cauchy e e´, portanto,
convergente.
10
(b) Se |xn+2 − xn+1| ≤ c |xn+1 − xn| para todo n ∈ N com 0 ≤ c < 1, enta˜o
vn = |x2 − x1|+ |x3 − x2|+ |x4 − x3|+ · · ·+ |xn+1 − xn|
≤ |x2 − x1|+ c |x2 − x1|+ c |x3 − x2|+ · · ·+ c |xn − xn−1|
≤ |x2 − x1|+ c |x2 − x1|+ c2 |x2 − x1|+ · · ·+ c2 |xn−1 − xn−2|
≤ · · · ≤ |x2 − x1|+ c |x2 − x1|+ c2 |x2 − x1|+ · · ·+ cn−1 |x2 − x1|
= |x2 − x1|
n−1∑
i=0
ci ≤ |x2 − x1|
∞∑
i=0
ci =
|x2 − x1|
1− c
para todo n ∈ N e (xn) tem, portanto, variac¸a˜o limitada.
(c) Defina
y1 =
x1
2
z1 = −x1
2
y2 =
|x2 − x1|+ x2
2
z2 =
|x2 − x1| − x2
2
y3 =
|x3 − x2|+ |x2 − x1|+ x3
2
z3 =
|x3 − x2|+ |x2 − x1| − x3
2
...
...
yn =
1
2
(
n−1∑
i=1
|xi+1 − xi|+ xn
)
zn =
1
2
(
n−1∑
i=1
|xi+1 − xi| − xn
)
=
vn−1 + xn
2
=
vn−1 − xn
2
.
Note que, para todo n ∈ N, vale xn = yn − zn. Mostremos agora que as sequeˆncias (yn) e
(zn) acima definidas sa˜o na˜o-decrescentes. Temos que
yn+1 − yn = 1
2
(
n∑
i=1
|xi+1 − xi|+ xn+1 −
n−1∑
i=1
|xi+1 − xi|+ xn
)
=
1
2
(
|xn+1 − xn|+
n−1∑
i=1
|xi+1 + xi|+ xn+1 −
n−1∑
i=1
|xi+1 − xi| − xn
)
=
1
2
[|xn+1 − xi|+ (xn+1 − xn)] ≥ 0.
De modo ana´logo, verificmos que (zn) e´ na˜o-decrescente. Logo, se (xn) tem varic¸a˜o limitada
temos que tanto (xn) quanto (vn) sa˜o convergentes e, portanto, sa˜o tambe´m convergentes e,
por conseguinte limitadas, as sequeˆncias (yn) e (zn). Por outro lado,se as sequeˆncias (yn) e
(zn) sa˜o limitadas enta˜o,devido ao fato de serem mono´tonas segue que elas sa˜o convergentes.
Como vn = yn+1 + zn+1 segue da´ı que (vn) e´ convergente e, consequentemente, limitada e,
deste modo (xn) tem variac¸a˜o limitada.
11
(d) Considere a sequeˆncia (xn) dada por xn =
(−1)n
n
. Temos que xn → 0, mas
|xn+1 − xn| =
∣∣∣∣∣(−1)
n+1
n+ 1
− (−1)
n
n
∣∣∣∣∣ =
∣∣∣∣∣(−1)
n+1
n+ 1
+
(−1)n+1
n
∣∣∣∣∣
=
∣∣∣∣ 1n+ 1 + 1n
∣∣∣∣ ≥ 1n .
Logo,
vn ≥
n∑
i=1
1
n
.
Como a se´rie harmoˆnica diverge segue que (vn) e´ ilimitada e, consequentemente, (xn) na˜o
possui variac¸a˜o limitada. �
20. Seja x1 = 1 e ponha xn+1 = 1 +
1
xn
. Verifique |xn+2 − xn+1| ≤ 1
2
|xn+1 − xn|. Conclua
que existe a = lim xn e determine a.
SOLUC¸A˜O: Da definic¸a˜o da sequeˆncia segue que xn ≥ 1, para todo n ∈ N. Assim, para todo
n ∈ N,
|xn+2 − xn+1| =
∣∣∣∣1 + 1xn+1 − 1−
1
xn
∣∣∣∣ =
∣∣∣∣ 1xn+1 −
1
xn
∣∣∣∣
=
∣∣∣∣xn − xn+1xnxn+1
∣∣∣∣ = |xn+1 − xn|
xn
(
1 +
1
xn
)
=
|xn+1 − xn|
xn + 1
≤ |xn+1 − xn|
2
.
Do Exerc´ıcio 19 segue que (xn) tem variac¸a˜o limitada e e´, portanto, uma sequeˆncia conver-
gente. Seja a = lim
n→∞
xn. Enta˜o a = lim
n→∞
xn−1. Temos enta˜o que
a = lim xn = lim
(
1− 1
xn−1
)
= 1− 1
lim xn−1
= 1− 1
a
,
isto e´, a satisfaz a igualdade
a = 1− 1
a
de onde extra´ımos a equac¸a˜o
λ2 − λ+ 1 = 0
que possui como soluc¸a˜o a =
1 +
√
5
2
. Assim,
lim xn =
1 +
√
5
2
.
�
12
21. Ponha x1 = 1 e defina xn = 1 +
√
xn. Mostre que a sequeˆncia (xn), assim obtida, e´
limitada. Determine a = lim xn.
SOLUC¸A˜O: Mostremos, utilizando induc¸a˜o em n que xn < 3, para todo n ∈ N. Para n = 1 o
resultado e´ o´bvio. Suponha a validade do resultado para n ∈ N e mostremos a validade do
mesmo para n+ 1. Temos que
xn+1 = 1 +
√
xn < 1 +
√
3 < 1 +
√
4 = 3.
Logo, a sequeˆncia (xn) e´ limitada. Novamente, utilizando induc¸a˜o, mostraremos que (xn) e´
uma sequeˆncia crescente, isto e´, que para todo n ∈ N xn+1 − xn > 0. Para n = 1 temos que
x2 − x1 = 2− 1 = 1 > 0.
Suponhamos a validade desta relac¸a˜o para n ∈ N e mostremos a validade da mesma para
n+ 1. Temos que
xn+2 − xn+1 = 1 +√xn+1 − 1−√xn = √xn+1 −√xn > 0,
pois, da hipo´tese de induc¸a˜o, xn+1 > xn e, consequentemente,
√
xn+1 >
√
xn. Logo (xn) e´
uma sequeˆncia crescente. Conclu´ımos enta˜o que (xn) e´ uma sequeˆncia mono´tona limitdae,
portanto, convergente. Seja a = lim
n→∞
xn. Enta˜o a = lim
n→∞
xn−1. Assim,
a = lim
n→∞
xn = lim
n→∞
(1 +
√
xn−1) = 1 +
√
a.
Logo a satisfaz a igualdade
a = 1 +
√
a,
isto e´,
a−√a− 1 = 0
Fazendo
√
a = λ, obtemos
λ2 − λ− 1 = 0,
de onde obtemos λ =
1 +
√
5
2
. Consequentemente
a = λ2 =
(
1 +
√
5
2
)2
=
3 +
√
5
2
.
�
22. A fim de que a sequeˆncia (xn) na˜o possua subsequeˆncia convergente e´ necessa´rio e
suficiente que lim |xn| = +∞.
SOLUC¸A˜O: Se (xn) na˜o possui nenhuma subsequeˆncia convergente enta˜o toda subsequeˆncia
de (xn) e´ ilimitada. Com efeito, se (xnk) e´ uma subsequeˆncia limitada de (xn) enta˜o, como
toda sequeˆncia possui uma subsequeˆncia mono´tona (Exerc´ıcio 15), ter´ıamos que (xnk) teria
uma subsequeˆncia mono´tona e limitada e, deste modo, tal sequeˆncia seria uma subsequeˆncia
13
convergente de (xn). Logo lim |xn| = +∞. Reciprocamente, se lim |xn| = +∞ enta˜o, dado
A > 0, existe n0 ∈ N tal que, sempre que n > n0,
|xn| > A.
Portanto, se (xnk) e´ uma subsequeˆncia de (xn), enta˜o existe k0, tal que, nk0 ≥ n0 e, portanto
|xnk | > A
sempre que k > k0. Temos, portanto, que a subsequeˆncia (xnk) e´ ilimitada e, por conseguinte,
divergente. �
23. Seja ϕ : N→ N uma sequeˆncia de nu´meros naturais. Prove que as seguintes afirmac¸o˜es
sa˜o equivalentes:
a) lim
n→∞
ϕ (n) = +∞;
b) Para todo k ∈ N, ϕ−1 (k) e´ um subconjunto finito de N;
c) Para todo conjunto finito F ⊂ N, ϕ−1 (F ) e´ finito.
Em particular, se ϕ : N→ N for injetiva, enta˜o lim
n→∞
ϕ (n) = +∞.
SOLUC¸A˜O: (a) ⇒ (b) : Suponha que lim
n→∞
ϕ (n) = +∞ e seja k ∈ N. Enta˜o existe n0 ∈ N
tal que
xn > k, ∀n > n0.
Logo ϕ−1 (k) ⊂ {n ∈ N; ϕ (n) ≤ k} ⊂ {1, 2, . . . , n0} e ϕ−1 (k) e´ um conjunto finito.
(b)⇒ (c) : Suponha que, para todo k ∈ N, ϕ−1 (k) seja finito. Seja F = {k1 < · · · < kn}
um subconjunto finito de N. Neste caso
ϕ−1 (F ) =
n⋃
i=1
ϕ−1 (ki) .
Logo ϕ−1 (F ) uma unia˜o finita de conjuntos finitos e, consequentemente, ϕ−1 (F ) e´ finito.
(c) ⇒ (a) : Suponha que para todo conjunto finito F ⊂ N, ϕ−1 (F ) seja finito. Para
todo k ∈ N, seja Ik = {1, . . . , k}. Enta˜o ϕ−1 (Ik) e´ finito para todo k ∈ N. Fazendo
n0 = maxϕ
−1 (Ik), enta˜o, para todo n > n0, temos que ϕ (n) /∈ Ik, ou seja,
ϕ (n) > k, ∀n > n0.
Logo, lim
n→∞
ϕ (n) = +∞. �
24. Seja (xn) uma sequeˆncia de nu´meros reais e suponha que ϕ : N → N cumpre uma (e
portanto todas as) condic¸o˜es do exerc´ıcio anterior. Prove que se lim xn = a e yn = xϕ(n),
enta˜o lim yn = a. Deˆ exemplo de ϕ : N → N sobrejetiva, tal que lim xn = a, mas na˜o vale
lim yn = a, onde yn = xϕ(n).
14
SOLUC¸A˜O: Sejam (xn) uma sequeˆncia de nu´meros reais tal que lim xn = a, ϕ : N → N uma
sequeˆncia de nu´meros naturais tal que limϕ (n) = +∞ e (yn) dada por yn = xϕ(n). Assim,
dado ε > 0, existe n0 ∈ N tal que
|xn − a| < ε
sempre que n > n0. Seja In0 = {1, . . . , n0}. Do exerc´ıcio anteriorsegue que ϕ−1 (In0) e´ um
conjunto finito. Seja n1 = maxϕ
−1 (In0). Deste modo, se n > n1 temos que ϕ (n) /∈ In0 , e,
consequentemente, ϕ (n) > n0. Portanto, sempre que n > n1
|yn − a| =
∣∣xϕ(n) − a∣∣ < ε
e lim yn = a.
Considere (xn) definida por xn =
1
n
. Enta˜o lim xn = 0. Seja ϕ : N→ N definida por
ϕ (n) =
{
1, se n e´ ı´mpar;
n
2
, se n e´ par.
Observe que ϕ e´ sobrejetiva. Fazendo yn = xϕ(n), observamos que a sequeˆncia assim definida
e´ divergente, pois a subsequeˆncia (y2n−1)n∈N definida por y2n−1 = x1 = 1 converge para 1
emquanto a subsequeˆncia (y2n)n∈N definida por y2n = xn =
1
n
converge para 0. �
25. Seja xn 6= 0 para todo n ∈ N. Se existirem n0 ∈ N e c ∈ R tais que 0 <
∣∣∣∣xn+1xn
∣∣∣∣ ≤ c < 1
para todo n > n0, enta˜o lim xn = 0. Se pore´m
∣∣∣∣xn+1xn
∣∣∣∣ ≥ c > 1 para todo n > n0, enta˜o
lim |xn| = +∞. Como aplicac¸a˜o, reobtenha os Exemplos 21 e 22 e mostre que lim n!
nn
= 0.
SOLUC¸A˜O: Seja xn 6= 0 para todo n ∈ N e suponha que existam n0 ∈ N e c ∈ R tais que
0 <
∣∣∣∣xn+1xn
∣∣∣∣ ≤ c < 1 para todo n > n0. Neste caso,
0 < |xn+1| ≤ c |xn| ≤ c2 |xn−1| ≤ · · · ≤ cn−n0 |xn0+1| .
Uma vez
lim
n→∞
cn−n0 |xn0+1| = |xn0+1| lim
n→∞
cn−n0 = 0,
segue do Teorema do Confronto que lim |xn| = 0 e, consequentemente, lim xn = 0. Se pore´m∣∣∣∣xn+1xn
∣∣∣∣ ≥ c > 1 para todo n > n0, enta˜o
|xn+1| ≥ c |xn| ≥ · · · ≥ cn−n0 |xn0+1| .
Uma vez que
lim
n→∞
cn−n0 |xn0+1| = |xn0+1| lim
n→∞
cn−n0 = +∞,
segue novamente do Teorema do Confronto que lim |xn| = +∞.
15
Reobtenhamos agora os Exemplos 21 e 22.
Exemplo 21. Seja a > 1. Enta˜o lim
an
n
= +∞. Com efeito,
∣∣∣∣an+1/ (n+ 1)an/n
∣∣∣∣ = an+1n+ 1 · nan = a · nn+ 1
e, uma vez que
lim a · n
n+ 1
= a lim
(
1− 1
n+ 1
)
= a > 1,
segue do Teorema da Conservac¸a˜o do Sinal que, se 1 < c < a, enta˜o existe n0 ∈ N tal
que a · n
n+ 1
≥ c sempre que n > n0. Portanto, do que acabamos de provar segue que
lim
an
n
= +∞. �
Exemplo 22. Para todo nu´mero real a > 0 tem-se lim
n!
an
= +∞. Com efeito,
∣∣∣∣(n+ 1)!/an+1n!/an
∣∣∣∣ = (n+ 1)!an+1 · a
n
n!
=
n+ 1
a
.
Como N e´ ilimitado superiormente, existe n0 ∈ N tal que n0 + 1 > a. Da´ı, fazendo c =
n0 + 1
a
> 1 segue que ∣∣∣∣(n+ 1)!/an+1n!/an
∣∣∣∣ = n+ 1a > c,
para todo n > n0. Logo, lim
n!
an
= +∞. �
Mostremos agora que lim
n!
nn
= 0. De fato,
∣∣∣∣∣(n+ 1)!/ (n+ 1)
n+1
n!/nn
∣∣∣∣∣ = (n+ 1)!(n+ 1)n+1 · n
n
n!
=
(
n
n+ 1
)n
=
(
1− 1
n+ 1
)n
<
(
1− 1
n
)n
.
Como lim
(
1− 1
n
)n
=
1
e
< 1 segue do Teorema da Conservac¸a˜o do Sinal que, se
1
e
< c < 1,
enta˜o existe n0 ∈ N tal que ∣∣∣∣∣(n+ 1)!/ (n+ 1)
n+1
n!/nn
∣∣∣∣∣ < c.
Logo, lim
n!
nn
= 0. �
16
26. Seja T um arranjo triangular de nu´meros na˜o-negativos,
t11
t21 t22
t31 t32 t33
· · · · · · · · · · · · · · · · · ·
tn1 tn2 tn3 · · · tnn
· · · · · · · · · · · · · · · · · ·
Fac¸a duas hipo´teses sobre o arranjo T . Primeira: cada linha tem soma igual a 1. Segunda:
cada coluna tem limite zero: lim
n→∞
tni = 0, para todo i ∈ N. Dada uma sequeˆncia convergente
(xn), com lim xn = a, use o arranjo T para transforma´-la numa sequeˆncia (yn), com
yn = tn1x1 + tn2x2 + · · ·+ tnnxn.
Prove que lim yn = a.
[Sugesta˜o: Considere inicialmente o caso a = 0. Dado ε > 0, existe p ∈ N tal que n > p ⇒
|xn| < ε
2
. Existe tambe´m A > 0, tal que |xn| < A para todo n. Em seguida, obtenha q ∈ N,
tal que n > q ⇒ |tn1| < δ, |tn2| < δ, . . ., |tnp| < δ, onde δ = ε
2pA
. Tome n0 = max {p, q}.
Observe que n > n0 ⇒ |yn| ≤ tn1 |x1|+· · ·+tnp |xp|+· · ·+tnn |xn|, onde a soma das p primeiras
parcelas na˜o excede
ε
2
e a soma das n−p parcelas restantes na˜o supera (tn,p+1 + · · ·+ tnn) ε
2
.
Logo |yn| < ε. O caso geral reduz-se imediatamente a este.]
SOLUC¸A˜O: Consideremos incialmente o caso a = 0. Assim, dado ε > 0, existe p ∈ N > 0 tal
que |xn| < ε
2
, sempre que n > p. Como (xn) e´ convergente, temos que e´ tambe´m limitada.
Neste caso, existe A > 0 tal que |xn| < A, para todo n ∈ N. Uma vez que as sequeˆncias
(tn1)
∞
n=1, (tn2)
∞
n=2, . . ., (tnp)
∞
n=p convergem para 0, tomando δ =
ε
2pA
, segue que existe q ∈ N
tal que |tn1| < δ, |tn2| < δ, . . ., |tnp| < δ, sempre que n > q. Tome n0 = max {p, q}. Assim,
|yn| = |tn1x1 + tn2x2 + · · ·+ tnnxn|
≤ tn1 |x1|+ tn2 |x2|+ · · ·+ tnp |xnp|+ tn,p+1 |xn,p+1|+ · · ·+ tnn |xn|
≤ tn1A+ tn2A+ · · ·+ tnpA+ tn,p+1 ε
2
+ · · ·+ tnn ε
2
= A (tn1 + · · ·+ tnp) + ε
2
(tn,p+1 + · · ·+ tnn)
≤ Apδ + ε
2
= Ap
ε
2Ap
+
ε
2
= ε,
sempre que n > n0. Logo lim yn = 0. O caso geral reduz-se imediatemente a este, pois se
xn → a 6= 0, enta˜o xn − a→ 0 e, portanto, do que acabamos de provar segue que
0 = lim [tn1 (x1 − a) + · · ·+ tnn (xn − a)]
= lim (tn1x1 − tn1a+ · · ·+ tnnxn − tnna)
= lim [tn1x1 + · · · tnnxn − a (tn1 + · · ·+ tnn)]
= lim (tn1x1 + · · ·+ tnnxn − a) ,
17
ou seja,
lim (tn1x1 + · · ·+ tnnxn) = a.
�
27. Se lim xn = a, pondo yn =
x1 + · · ·+ xn
n
, tem-se ainda lim yn = a.
[Sugesta˜o: Use o exerc´ıcio anterior.]
SOLUC¸A˜O: Considere o seguinte arranjo triangular
1
1
2
1
2
1
3
1
3
1
3
· · · · · · · · · · · · · · ·
1
n
1
n
1
n
· · · 1
n
· · · · · · · · · · · · · · ·
Temos qua cada linha deste arranjo tem soma igual a 1 e cada coluna tem limite zero. Deste
modo, segue do exerc´ıcio anterior que a sequeˆncia (yn) dada por
yn =
x1 + · · ·+ xn
n
=
1
n
x1 + · · ·+ 1
n
xn
tem limite a. �
28. Se lim xn = a e os xn sa˜o todos positivos, enta˜o lim n
√
x1x2 · · · xn = a.
[Sugesta˜o: Tome logar´ıtmos e reduza ao problema anterior.] Conclua que se an > 0 e
lim
an+1
an
= a enta˜o lim n
√
an = a.
SOLUC¸A˜O I: Se a = 0, segue da desigualdade entre as me´dias aritme´tica e geome´trica que
0 ≤ n√x1x2 · · · xn ≤ x1 + x2 + · · ·+ xn
n
.
Do exerc´ıcio anterior segue que
lim
x1 + x2 + · · ·+ xn
n
= 0
e, portanto, do Teorema do Confronto, segue que
lim n
√
x1x2 · · · xn = 0.
Se a > 0, enta˜o uma vez que f (x) = log x e´ uma func¸a˜o cont´ınua que possui inversa cont´ınua
segue xn → a⇔ log xn → log a. Deste modo
log n
√
x1x2 · · · xn = log (x1x2 · · · xn)
1
2 =
1
n
log (x1x2 · · · xn)
=
1
n
(log x1 + log x2 + · · ·+ log xn) = log a.
18
Logo, lim n
√
x1x2 · · · xn = a.
Seja agora (an) uma sequeˆncia tal que an > 0 para todo n ∈ N e lim an+1
an
= a. Fazendo
x1 = a1 e xn+1 =
an+1
an
temos que
a = lim n
√
x1x2 · · · xn = lim n
√
a1
a2
a1
· · · an
an−1
= lim n
√
an.
�
SOLUC¸A˜O II: Se a = 0, segue da desigualdade entre as me´dias aritme´tica e geome´trica que
0 ≤ n√x1x2 · · · xn ≤ x1 + x2 + · · ·+ xn
n
.
Do exerc´ıcio anterior segue que
lim
x1 + x2 + · · ·+ xn
n
= 0
e, portanto, do Teorema do Confronto, segue que
lim n
√
x1x2 · · · xn = 0.
Se a > 0 seja yn a me´dia harmoˆnica dos termos x1, . . . , xn, isto e´,
yn =
n
1
x1
+
1
x2
+ · · ·+ 1
xn
.
Se (xn) converge para a enta˜o (1/xn) converge para
1
a
. Deste modo, segue do Exerc´ıcio 27
que
lim
1
x1
+
1
x2
+ · · ·+ 1
xn
n
= a.
Logo,
lim yn = lim
n
1
x1
+
1
x2
+ · · ·+ 1
xn
= lim
1
1
x1
+
1
x2
+ · · ·+ 1
xn
n
=
1
lim
1
x1
+
1
x2
+ · · ·+ 1
xn
n
=
1
1
a
= a.
Da desigualdade entre as me´dias harmoˆnica, geome´trica e aritme´tica temos que
n
1
x1
+
1
x2
+ · · ·+ 1
xn
≤ n√x1x2 · · · xn ≤ x1 + x2 + · · ·+ xn
n
19
e do Teorema do Confronto segue o resultado. �
29. Seja yn > 0 para todo n ∈ N, com
∑
yn = +∞. Se lim xn
yn
= a enta˜o
lim
x1 + x2 + · · ·+ xn
y1 + y2 + · · ·+ yn =a.
SOLUC¸A˜O: Considere o arranjo triangular
t11
t21 t22
t31 t32 t33
· · · · · · · · · · · · · · · · · ·
tn1 tn2 tn3 · · · tnn
· · · · · · · · · · · · · · · · · ·
tal que tnj =
yj
y1 + y2 + · · ·+ yn , j = 1, . . . , n. Enta˜o, cada linha do arranjo tem soma igual
a 1 e cada coluna tem limite zero. Portanto, segue do Exerc´ıcio 26 que
lim
x1 + x2 + · · ·+ xn
y1 + y2 + · · ·+ yn = lim
[
y1
y1 + y2 + · · ·+ yn
x1
y1
+ · · ·+ yn
y1 + y2 + · · ·+ yn
x1
yn
]
= lim
[
tn1
x1
y1
+ tn2
x2
y2
+ · · ·+ tnnxn
yn
]
= a.
�
30. Se (yn) e´ crescente e lim yn = +∞, enta˜o lim xn+1 − xn
yn+1 − yn = a ⇒ lim
xn
yn
= a. (Use o
Exerc´ıcio 29.)
SOLUC¸A˜O: Fazendo v1 = x1 e vn+1 = xn+1 − xn, w1 = y1 e wn = yn+1 − yn, temos que
lim
vn
wn
= a. Do Exerc´ıcio 29 segue que
a = lim
vn
wn
= lim
v1 + v2 + · · ·+ vn
w1 + w2 + · · ·+ wn = lim
xn − xn−1 + · · ·+ x2 − x1 + x1
yn − yn−1 + · · ·+ y2 − y1 + y1 = lim
xn
yn
.
�
31. lim
n→∞
1p + 2p + · · ·+ np
np+1
=
1
p+ 1
. (Use o exerc´ıcio anterior)
20
SOLUC¸A˜O: Fazendo xn = 1
p + 2p + · · ·+ np e yn = np+1, temos que
lim
n→∞
xn+1 − xn
yn+1 − yn = limn→∞
(n+ 1)p
(n+ 1)p+1 − np+1 = limn→∞
(n+ 1)p
p+1∑
k=1
(
p+1
k
)
nk − np+1
= lim
n→∞
(n+ 1)p
p∑
k=1
(
p+1
k
)
nk
= lim
n→∞
np
(
1 +
1
n
)p
np
p∑
k=1
(
p+1
k
)
nk−p
= lim
n→∞
(
1 +
1
n
)p
p∑
k=1
(
p+1
k
) 1
np−k
= lim
n→∞
(
1 +
1
n
)p
(
p+1
p
)
+
p−1∑
k=1
(
p+1
k
) 1
np−k
=
lim
n→∞
(
1 +
1
n
)p
(
p+1
p
)
+
p−1∑
k=1
(
p+1
k
)
lim
n→∞
1
np−k
=
1(
p+1
p
) = 1
p+ 1
.
Logo, do exerc´ıcio anterior segue que
1
p+ 1
= lim
n→∞
xn+1 − xn
yn+1 − yn = limn→∞
xn
yn
= lim
n→∞
1p + 2p + · · ·+ np
np+1
.
�
32. Para todo n ∈ N, tem-se 0 < e −
(
1 +
1
1!
+
1
2!
+ · · ·+ 1
n!
)
<
1
n!n
. Conclua da´ı que o
nu´mero e e´ irracional.
SOLUC¸A˜O: Para todo n ∈ N temos que
n!
[
e−
(
1 +
1
1!
+
1
2!
+ · · ·+ 1
n!
)]
= n!
[
∞∑
k=0
1
k!
−
n∑
k=0
1
k!
]
= n!
∞∑
k=n+1
1
k!
=
∞∑
k=n+1
n!
k!
.
Se k > n, enta˜o
n!
k!
=
n!
n! (n+ 1) (n+ 2) · · · (n+ (k − n))
=
1
(n+ 1) (n+ 2) · · · (n+ (k − n))
≤ 1
(n+ 1)k−n
.
21
Note que se k > n+ 1 enta˜o vale a desigualdade estrita. Deste modo
n!
[
e−
(
1 +
1
1!
+
1
2!
+ · · ·+ 1
n!
)]
=
∞∑
k=n+1
n!
k!
<
∞∑
k=n+1
1
(n+ 1)k−n
=
1
n+ 1
∞∑
k=0
1
(n+ 1)k
=
1
n+ 1
1
1− 1
n+ 1
=
1
n
.
Logo, e −
(
1 +
1
1!
+
1
2!
+ · · ·+ 1
n!
)
<
1
n!n
. Supondo e racional segue que existem inteiros
positivos p e q tais que e =
p
q
. Note que devemos ter q > 1, pois, caso contra´rio, e seria um
nu´mero inteiro tal que 2 < e < 3. Assim, do que provamos assima segue que
0 <
p
q
−
(
1 +
1
1!
+
1
2!
+ · · ·+ 1
q!
)
<
1
q!q
.
Multiplicando todos os membros da desigualdade acima por q! segue que
0 < p (q − 1!)−
q∑
k=0
q!
k!
<
1
q
< 1
o que e´ um absurdo, pois o termo p (q − 1!)−
q∑
k=0
q!
k!
∈ Z. Logo e e´ irracional. �
33. lim
n→∞
1
n
n
√
(n+ 1) (n+ 2) · · · 2n = 4
e
. (Use o final do Exerc´ıcio 28.)
SOLUC¸A˜O: Fazendo an =
(n+ 1) (n+ 2) · · · 2n
nn
=
2n!
n!nn
temos que
lim
n→∞
an+1
an
= lim
n→∞
(2n+ 2)!
(n+ 1)! (n+ 1)n+1
· n!n
n
2n!
= lim
n→∞
(2n+ 2) (2n+ 1)nn
(n+ 1)n+2
= lim
n→∞
2nn (2n+ 1)
(n+ 1)n+1
= lim
n→∞
2nn+1
(
2 +
1
n
)
nn+1
(
1 +
1
n
)n+1 = limn→∞
(
4 +
2
n
)
(
1 +
1
n
)
·
(
1 +
1
n
)n
=
lim
n→∞
(
4 +
2
n
)
lim
n→∞
(
1 +
1
n
)
lim
n→∞
(
1 +
1
n
)n+1 = 4e .
22
Do Exerc´ıcio 28 segue que
lim
n→∞
1
n
n
√
(n+ 1) (n+ 2) · · · 2n = lim
n→∞
n
√
an = lim
n→∞
an+1
an
=
4
e
.
�
34. Prove que se definirmos an pela igualdade n! = n
ne−nan, teremos lim n
√
an = 1.
SOLUC¸A˜O: Se an =
n!en
nn
temos que
lim
an+1
an
= lim
(n+ 1)!en+1
(n+ 1)n+1
· n
n
n!en
= lim
nne
(n+ 1)n
= lim
nne
nn
(
1 +
1
n
)n
=
e
lim
(
1 +
1
n
)n = e
e
= 1.
Do Exerc´ıcio 28 segue que
lim n
√
an = lim
an+1
an
= 1.
�
35. Sejam
∑
an e
∑
bn se´ries de termos positivos. Se
∑
bn = +∞ e existe n0 ∈ N tal que
an+1
an
≥ bn+1
bn
para todo n > n0 enta˜o
∑
an = +∞.
SOLUC¸A˜O: Para todo n > n0 temos que
bn =
bn
bn−1
bn−1
bn−2
· · · bn0+2
bn0+1
bn0+1 ≤
an
an−1
an−1
an−2
· · · an0+2
an0+1
bn0+1 =
bn0+1
an0+1
an.
De
∞∑
n=1
bn = +∞ segue que
∞∑
n=n0+1
bn = +∞. Mas
∞∑
n=n0+1
bn ≤
∞∑
n=n0+1
bn0+1
an0+1
an =
bn0+1
an0+1
∞∑
n=n0+1
an ≤ bn0+1
an0+1
∞∑
n=1
an
e, portanto, do teste de comparac¸a˜o segue que
bn0+1
an0+1
∞∑
n=1
an = +∞ e, consequentemente,
∞∑
n=1
an = +∞. �
36. Sejam
∑
an e
∑
bn se´ries de termos positivos. Se lim
an
bn
= 0 e
∑
bn converge enta˜o
∑
an
converge. Se lim
an
bn
= c 6= 0, enta˜o ∑an coverge se, e somente se, ∑bn converge.
23
SOLUC¸A˜O: Se lim
an
bn
= 0, enta˜o a sequeˆncia
(
an
bn
)
n∈N
e´ limitada e, consequentemente, existe
K > 0 tal que
an
bn
≤ K, ∀n ∈ N
ou seja,
an ≤ Kbn, ∀n ∈ N.
Deste modo,
0 <
∞∑
n=1
an ≤
∞∑
n=1
Kbn = K
∞∑
n=1
bn < +∞
e, portanto,
∑
an e´ convergente.
De modo ana´logo, se lim
an
bn
= c 6= 0, enta˜o lim bn
an
= lim
1(
an
bn
) = 1
c
e as sequeˆncias
(
an
bn
)
n∈N
e
(
bn
an
)
n∈N
sa˜o limitadas, isto e´, existem K,L > 0 tais que
an
bn
≤ K e bn
an
≤ L, ∀n ∈ N,
ou seja,
an ≤ Kbn e bn ≤ Lan, ∀n ∈ N.
Deste modo,
0 <
∞∑
n=1
an ≤ K
∞∑
n=1
bn ≤ KL
∞∑
n=1
an
e, consequentemente, segue do teste de comparac¸a˜o que
∑
an coverge se, e somente se,
∑
bn
converge. �
37. Para todo polinoˆmio p (x) de grau superior a 1, a se´rie
∑ 1
p (n)
converge.
SOLUC¸A˜O: Sabemos que se m > 1, enta˜o
∞∑
n=1
1
nm
converge. Seja p (x) = a0+a1x+ · · ·+amxm
um polinoˆmio de grau m > 1 tal que am > 0. Considerando a sequeˆncia
xn =
a0
nm
+
a1
nm−1
+ · · ·+ am−1
n
+ am,
temos que lim xn = am > 0. Seja 0 < c < am. Do Teorema da Conservac¸a˜o do Sinal segue
que existe n0 ∈ N tal que
a0
nm
+
a1
nm−1
+ · · ·+ am−1
n
+ am > c,
24
para todo n > n0. Deste modo,
∞∑
n=1
∣∣∣∣ 1p (n)
∣∣∣∣ = n0∑
n=1
∣∣∣∣ 1p (n)
∣∣∣∣+ ∞∑
n=n0+1
∣∣∣∣ 1p (n)
∣∣∣∣
=
n0∑
n=1
∣∣∣∣ 1p (n)
∣∣∣∣+ ∞∑
n=n0+1
1
nm
( a0
nm
+
a1
nm−1
+ · · ·+ am−1
n
+ am
)
<
n0∑
n=1
∣∣∣∣ 1p (n)
∣∣∣∣+ ∞∑
n=n0+1
1
cnm
=
n0∑
n=1
∣∣∣∣ 1p (n)
∣∣∣∣+ 1c
∞∑
n=n0+1
1
nm
< +∞.
Assim,
∞∑
n=1
1
p (n)
e´ absolutamente convergente e, portanto, convergente. Se am < 0, enta˜o
∞∑
n=1
1
p (n)
= −
∞∑
n=1
1
−p (n)
e, consequentemente, do que acabamos de mostrar segue o resultado. �
38. Se −1 < x < 1 e (
m
n
)
=
m (m− 1) · · · (m− n+ 1)
n!
(1.6)
enta˜o lim
n→∞
(
m
n
)
xn = 0 para quaisquer m ∈ R e n ∈ N.
SOLUC¸A˜O: Se m = 0, na˜o ha´ o que fazer. Se m ∈ N, enta˜o quando n ≥ m+ 1 temos que
m (m− 1) · · · (m− n+ 1)
n!
= 0
pois o fator m − m = 0 fara´ parte do denominador neste caso. Se m /∈ N∪{0}, enta˜o o
numerador de (1.6) nunca e´ zero. Assim, fazendo an =
(
m
n
)
xn temos que
lim
n→∞
∣∣∣∣an+1an
∣∣∣∣ = limn→∞
∣∣∣∣∣∣∣∣
(m
n+ 1
)
xn+1(
m
n
)
xn
∣∣∣∣∣∣∣∣
= lim
n→∞
∣∣∣∣m (m− 1) · · · (m− n+ 1) (m− n) xn+1(n+ 1)! · n!m (m− 1) · · · (m− n+ 1) xn
∣∣∣∣
= lim
n→∞
∣∣∣∣(m− n) xn+ 1
∣∣∣∣ = limn→∞
∣∣∣∣∣∣∣
(m
n
− 1
)
1 +
1
n
x
∣∣∣∣∣∣∣ = |x| < 1.
Logo, do Exerc´ıcio 25 segue que lim
n→∞
(
m
n
)
xn = 0. �
25
39. Se a sequeˆncia (an) e´ na˜o-crescente e lim an = 0, o mesmo ocorre com bn =
a1 + · · ·+ an
n
.
Conclua que, neste caso, a se´rie a1 − 1
2
(a1 + a2) +
1
3
(a1 + a2 + a3)− · · · e´ convergente.
SOLUC¸A˜O: Uma vez que (an) e´ na˜o-crescente e lim an = 0, temos, para todo n ∈ N, que
an ≥ 0 e
an+1 ≤ an
an+1 ≤ an−1
...
an+1 ≤ a1.
Somando estas desigualdades membro a membro, obtemos que
nan+1 ≤ a1 + · · ·+ an,
e, portanto,
an+1 ≤ a1 + · · ·+ an
n
= bn.
Deste modo,
bn+1 =
a1 + · · ·+ an + an+1
n+ 1
=
1
n+ 1
[
n
(
a1 + · · ·+ an
n
)
+ an+1
]
=
1
n+ 1
(nbn + an+1) ≤ 1
n+ 1
(nbn + bn) = bn
e, consequentemente, a sequeˆncia (bn) e´ mono´tona na˜o-crescente. Assim, do Teorema de
Leibniz segue que a se´rie
a1 − 1
2
(a1 + a2) +
1
3
(a1 + a2 + a3)− · · · =
∞∑
n=1
(−1)n−1 bn
converge. �
40. Prove que, para todo a ∈ R, a se´rie a2+ a
2
1 + a2
+
a2
(1 + a2)2
+ · · · e´ convergente e calcule
sua soma.
SOLUC¸A˜O: Se a = 0, enta˜o a se´rie converge para 0. Se a 6= 0, enta˜o a se´rie em questa˜o e´
uma se´rie geome´trica com raza˜o 0 < r =
1
1 + a2
< 1 e, portanto,
a2 +
a2
1 + a2
+
a2
(1 + a2)2
+ · · · = a2
∞∑
n=0
(
1
1 + a2
)n
= a2
1
1− 1
1 + a2
= 1 + a2.
�
26
41. Para todo p ∈ N fixado, a se´rie ∑ 1
n (n+ 1) · · · (n+ p) converge.
SOLUC¸A˜O: Considere o polinoˆmio P (x) = x (x+ 1) · · · (x+ p). Temos que, para todo p ∈ N,
o grau de P (x) e´ maior do que 1. Portanto, segue do Exerc´ıcio 37 que
∑ 1
n (n+ 1) · · · (n+ p) =
∑ 1
P (n)
converge. �
42. Se
∑
an converge e an > 0 enta˜o
∑
(an)
2 e
∑ an
1 + an
convergem.
SOLUC¸A˜O: Uma vez que
∑
an converge temos que lim an = 0. Portanto, existe n0 ∈ N tal
que 0 < an < 1 para todo n > n0 e, neste caso, (an)
2 < an, para todo n > n0. Assim,
∞∑
n=1
(an)
2 =
n0∑
n=1
(an)
2 +
∞∑
n=n0+1
(an)
2
<
n0∑
n=1
(an)
2 +
∞∑
n=n0+1
an < +∞.
Temos ainda que, para todo n ∈ N, 1 + an > 1 e, consequentemente,
1
1 + an
< 1, ∀n ∈ N.
Multiplicando ambos os membros desta desigualdade por an > 0, segue que
an
1 + an
< an, ∀n ∈ N.
Logo,
0 <
∑ an
1 + an
<
∑
an < +∞
e, do Teste de Comparac¸a˜o, segue a convergeˆncia de
∑ an
1 + an
. �
43. Se
∑
(an)
2 converge enta˜o
∑an
n
converge.
SOLUC¸A˜O: Uma vez que
∑
(an)
2 converge, segue que lim (an)
2 = lim |an| = 0. Sejam
N1 =
{
n ∈ N; |an| ≤ 1
n
}
e N2 =
{
n ∈ N; |an| > 1
n
}
.
Enta˜o |an|
n
≤ 1
n2
, ∀n ∈ N1
e |an|
n
< (an)
2 , ∀n ∈ N2.
27
Deste modo,
0 ≤∑∞n=1 ∣∣∣ann
∣∣∣ = ∞∑
n=1
|an|
n
=
∑
n∈N1
|an|
n
+
∑
n∈N2
|an|
n
≤ ∑
n∈N1
1
n2
+
∑
n∈N2
(an)
2 ≤∑∞n=1 1n2 +∑∞n=1 (an)2 < +∞.
Logo,
∑an
n
e´ absolutamente convergente e, portanto, convergente. �
44. Se (an) e´ decrescente e
∑
an converge enta˜o limn · an = 0.
SOLUC¸A˜O I: Uma vez que
∑
an e´ convergente temos que lim an = 0 e, como (an) e´ de-
crescente, temos que an > an+1 > 0 para todo n ∈ N. Deste modo, denotando por (sn) a
sequeˆncia das reduzidas de
∑
an, temos que
0 < na2n < an+1 + · · ·+ a2n < an+1 + · · ·+ a2n + · · · = s− sn,
para todo n ∈ N. Assim, como lim (s− sn) = 0, segue do Teorema do Confronto que
limna2n = 0 e, portanto,
lim 2nan = 2 limnan = 0.
Do mesmo modo,
0 < na2n−1 < an + · · ·+ a2n−1 < an + · · ·+ a2n−1 + · · · = s− sn−1,
para todo n ∈ N. Assim, como lim (s− sn−1) = 0, segue do Teorema do Confronto que
limna2n−1 = 0 e, portanto,
lim (2n− 1) a2n−1 = 2 limna2n−1 − lim a2n−1 = 0.
Do Exerc´ıcio 3 segue o resultado �
SOLUC¸A˜O II: Uma vez que
∑
an e´ convergente temos que lim an = 0 e, como (an) e´ de-
crescente, temos que an > an+1 > 0 para todo n ∈ N. Afirmamos que existe n0 ∈ N tal
que an <
1
n
, para todo n ≥ n0. Com efeito, se isto na˜o ocorresse, enta˜o, para todo n ∈ N
ter´ıamos an ≥ 1
n
e, consequentemente,
∑
an ≥
∑ 1
n
e da divergeˆncia da se´rie harmoˆnica seguiria, de acordo com o teste de comparac¸a˜o, a di-
vergeˆncia da se´rie
∑
an, contradizendo o fato de esta se´rie ser convergente. Nestas condic¸o˜es
afirmamos que existe ε > 0 tal que an ≤ 1
n1+ε
para todo n ≥ n0. De fato, caso isso na˜o
ocorresse, enta˜o dado ε > 0 ter´ıamos an >
1
n1+ε
, para todo n ≥ n0. Fazendo ε→ 0, ter´ıamos
enta˜o an ≥ 1
n
, contradizendo o que provamos anteriormente. Logo,
0 < nan <
1
nε
.
28
Como lim
1
nε
= 0, o resultado segue do Teorema do confronto. �
45. Se (an) e´ decrescente e
∑
an = +∞, enta˜o
lim
n→∞
a1 + a3 + · · ·+ a2n−1
a2 + a4 + · · ·+ a2n = 1.
SOLUC¸A˜O: Uma vez que (an) e´ decrescente e
∑
an = +∞ temos que an > 0, para todo
n ∈ N. Com efeito, se existisse n0 ∈ N tal que an0 ≤ 0, enta˜o an < 0, para todo n > n0.
Deste modo
∞∑
n=n0+1
an < 0 e, consequentemente,
∞∑
n=1
an =
n0∑
n=1
an +
∞∑
n=n0+1
an <
∑n0
n=1an < +∞,
o que e´ uma contradic¸a˜o. Temos ainda que
∞∑
n=1
an =
∞∑
n=1
a2n−1 +
∞∑
n=1
a2n (1.7)
e, como
∑
an = +∞, segue que pelo menos uma das parcelas em (1.7) vale +∞. Se for
∞∑
n=1
a2n = +∞, enta˜o, como a2n−1 < a2n para todo n ∈ N, teremos que
∞∑
n=1
a2n−1 = +∞. Por
outro lado, se
∞∑
n=1
a2n−1 = +∞, enta˜o,
∞∑
n=1
a2n+1 = −a1+
∞∑
n=1
a2n−1 = +∞ e, como, a2n > a2n+1
para todo n ∈ N, teremos
∞∑
n=1
a2n = +∞. Logo, temos que
∞∑
n=1
a2n−1 =
∞∑
n=1
a2n = +∞.
Observe agora que, para todo n ∈ N, temos que
a1 + a3 + · · ·+ a2n−1
a2 + a4 + · · ·+ a2n > 1
e
a3 + a5 + · · ·+ a2n−1
a2 + a4 + · · ·+ a2n < 1.
Assim,
1 <
a1 + a3 + · · ·+ a2n−1
a2 + a4 + · · ·+ a2n
=
a1
a2 + a4 + · · ·+ a2n +
a3 + a5 + · · ·+ a2n−1
a2 + a4 + · · ·+ a2n
<
a1
a2 + a4 + · · ·+ a2n + 1.
29
Uma vez que lim
a1
a2 + a4 + · · ·+ a2n = 0, segue do Teorema do Confronto que
lim
a1 + a3 + · · ·+ a2n−1
a2 + a4 + · · ·+ a2n = 1.
�
46. Seja (an) uma sequeˆncia na˜o-crescente, com lim an = 0. A se´rie
∑
an converge se, e
somente se,
∑
2na2n converge.
SOLUC¸A˜O: Uma vez que (an) e´ na˜o-crescente e lim an = 0, temos que
an ≥ an+1 ≥ 0,
para todo n ∈ N. Deste modo, supondo ∑an <∞ temos que
∞∑
n=1
an = a1 + a2 + (a3 + a4) + (a5 + a6 + a7 + a8) + · · ·
≥ a2 + 2a4 + 4a8 + · · · =
∞∑
n=1
2n−1a2n.
Logo,
∞∑
n=1
2na2n = 2
∞∑
n=1
2n−1a2n ≤ 2
∞∑
n=1
an <∞. (1.8)
Por outro lado, supondo que
∑
2na2n <∞, temos que
∞∑
n=1
an = a1 + (a2 + a3) + (a4 + a5 + a6 + a7) + · · ·
≤ a1 + 2a2 + 4a4 + · · · = a1 +
∞∑
n=1
2na2n ,
ou seja,
∞∑
n=1
an ≤ a1 +
∞∑
n=1
2na2n <∞. (1.9)
Logo, a se´rie
∑
an converge se, e somente se,
∑
2na2n converge. �
47. Prove que o conjunto dos valores de adereˆncia da sequeˆncia xn = cos (n) e´ o intervalo
fechado [−1, 1].
SOLUC¸A˜O: Da sobrejetividade da func¸a˜o cosseno segue que dado t ∈ [−1, 1], existe x ∈ R tal
que cos x = t. Uma vez que o conjunto
X = {2ppi + q; p, q ∈ Z}
e´ denso na reta, segue que existem sequeˆncias (pn) e (qn) tais que lim (2pipn + qn) = x. Da
continuidade e da paridade da func¸a˜o cosseno, temos que
t = cos x = cos (lim 2pipn + qn) = lim cos (2pipn + qn) = lim cos qn = lim cos (|qn|)
30
e t e´ limite de uma subsequeˆncia de xn = cosn. �
48. Sejam a, b nu´meros reais positivos. Defina indutivamente as sequeˆncias (xn), (yn) pondo
x1 =
√
ab, y1 =
a+ b
2
e xn+1 =√
xnyn, yn+1 =
xn + yn
2
. Prove que (xn) e (yn) convergem
para o mesmo limite, chamado a me´dia aritme´tico-geome´trica entre a e b.
SOLUC¸A˜O: Afirmamos que xn ≤ yn, para todo n ∈ N. Com efeito, da desigualdade entre as
me´dias aritme´tica e geome´trica segue que
x1 =
√
ab ≤ a+ b
2
= y1.
e, de modo geral,
xn+1 =
√
xnyn ≤ xn + yn
2
= yn+1.
Observe ainda que segue imediatamente deste fato que
yn+1 =
xn + yn
2
≤ yn + yn
2
= yn e xn+1 =
√
xnyn ≥ √xnxn = xn.
para todo n ∈ N e, ale´m disso,
0 ≤ xn ≤ yn ≤ y1,
para todo n ∈ N, ou seja, (xn) e (yn) sa˜o sequeˆncias mono´tonas limitadas e, portanto,
convergentes. Sejam
x = lim xn e y = lim yn.
Enta˜o
2y = 2 lim yn+1 = 2 lim
xn + yn
2
= lim xn + lim yn = x+ y,
isto e´,
x = y.
�
49. Sejam a1 ≥ a2 ≥ · · · ≥ 0 e sn = a1 − a2 + · · ·+ (−1)n−1 an. Prove que a sequeˆncia (sn)
e´ limitada e que lim sup sn − lim inf sn = lim an.
SOLUC¸A˜O: A sequeˆncia (an) e´ mono´tona limitda e, portanto, convergente. Observe que, do
fato de ser an ≥ an+1 e an ≥ 0, para todo n ∈ N, conclu´ımos que
s2n−1 = a1 − a2 + · · ·+ a2n−3 − a2n−2 + a2n−1
= (a1 − a2) + · · ·+ (a2n−3 − a2n−2) + a2n−1 ≥ 0
e
s2n = a1 − a2 + · · ·+ a2n−1 − a2n
= (a1 − a2) + · · ·+ (a2n−1 − a2n) ≥ 0
31
Note ainda que
s2n+1 − s2n−1 = −a2n + a2n+1 ≤ 0⇒ s2n−1 ≥ s2n+1
s2n+2 − s2n = a2n+1 − a2n+2 ≥ 0⇒ ssn+2 ≥ s2n
s2n+2k+1 − s2n = a2n+1 − a2n+2 + · · ·+ a2n+2k−1 − a2n+2k + a2n+2k+1
= (a2n+1 − a2n+2) + · · ·+ (a2n+2k−1 − a2n+2k) + a2n+2k+1
≥ 0⇒ s2n+2k+1 ≥ s2n.
Logo, para todo n ∈ N tem-se s1 ≥ sn ≥ 0, ou seja (sn) e´ uma sequeˆncia limitada. Ale´m
disso, por serem mono´tonas e limitadas, as sequeˆncias (s2n−1)n∈N e (s2n)n∈N sa˜o convergentes.
Fazendo
Sn = {sn, sn+1, . . .}
temos que
inf S2n−1 = inf {s2n−1, s2n, . . .} = s2n e supS2n−1 = sup {s2n−1, s2n, . . .} = s2n−1,
inf S2n = inf {s2n, s2n+1, . . .} = s2n e supS2n = sup {s2n, s2n+1, . . .} = s2n+1
e, portanto,
lim sup sn − lim inf sn = lim s2n−1 − lim s2n
= lim s2n+1 − lim s2n
= lim (s2n+1 − s2n)
= lim a2n+1 = lim an.
A u´ltima igualdade acima decorre do fato de a sequeˆncia (an) ser convergente. �
32
Cap´ıtulo 2
Topologia da Reta
1. Um conjunto A ⊂ R e´ aberto se, e somente se, cumpre a seguinte condic¸a˜o: “se uma
sequeˆncia (xn) converge para um ponto a ∈ A, enta˜o xn ∈ A para todo n suficientemente
grande.
SOLUC¸A˜O: Suponha que A ⊂ R seja aberto e seja a ∈ A. Enta˜o existe ε > 0 tal que
(a− ε, a+ ε) ⊂ A. Se (xn) e´ uma sequeˆncia que converge para a, enta˜o existe n0 ∈ N tal
que
xn ∈ (a− ε, a+ ε) ⊂ A, ∀n ≥ n0,
ou seja, xn ∈ A para todo n suficientemente grande. Reciprocamente, suponha que A ⊂ R
tem a propriedade de que se uma sequeˆncia (xn) converge para um ponto a ∈ A, enta˜o
xn ∈ A para todo n suficientemente grande. Suponha, por absurdo, que A 6= int(A). Enta˜o
existe a ∈ A tal que a /∈ int(A). Logo, a ∈ ∂A = ∂Ac e, consequentemente, a ∈ Ac. Deste
modo, existe uma sequeˆncia (xn) de pontos de A
c que converge para a. Por hipo´tese, segue
que xn ∈ A para todo n suficientemente grande e, consequentemente, A ∩ Ac 6= ∅, o que e´
uma contradic¸a˜o. Portanto A = int(A) e A e´ aberto. �
2. Tem-se lim xn = a se, e somente se, para todo aberto A contendo o ponto a, existe n0 ∈ N
tal que n > n0 implica xn ∈ A.
SOLUC¸A˜O: Suponha que lim xn = a e seja A um aberto que conte´m a. Enta˜o existe ε > 0
tal que (a− ε, a+ ε) ⊂ A. Deste modo, existe tambe´m n0 ∈ N tal que
xn ∈ (a− ε, a+ ε) ⊂ A,
sempre que n > n0, ou seja, existe n0 ∈ N tal que xn ∈ A sempre que n > n0. Reciproca-
mente, suponha que dado um aberto A contendo o ponto a existe n0 ∈ N tal que xn ∈ A
para todo n > n0. Enta˜o, dado ε > 0 o intervalo (a− ε, a+ ε) e´ um aberto que conte´m a e,
portanto, existe n0 ∈ N tal que xn ∈ (a− ε, a+ ε) sempre que n > n0, isto e´, lim xn = a. �
3. Seja B ⊂ R aberto. Enta˜o, para todo x ∈ R, o conjunto
x+ B = {x+ y; y ∈ B}
33
e´ aberto. Analogamente, se x 6= 0, enta˜o o conjunto
x · B = {x · y; y ∈ B}
e´ aberto.
SOLUC¸A˜O: Seja I = (a, b) um intervalo aberto. Enta˜o dado x ∈ R o conjunto x + I e´ o
intervalo aberto (a+ x, b+ x). Com efeito,
d ∈ (a+ x, b+ x)⇒ a+ x < d < b+ x⇒ a < d− x < b
⇒ d− x ∈ (a, b) = I ⇒ d = x+ (d− x) ∈ x+ I
⇒ (a+ x, b+ x) ⊂ x+ I
e, reciprocamente,
d ∈ x+ I ⇒ d = x+ y, para algum y ∈ I ⇒ d− x = y ∈ I
⇒ a < d− x < b⇒ a+ x < d < b+ x⇒ d ∈ (a+ x, b+ x)
⇒ x+ I ⊂ (a+ x, b+ x) .
Do mesmo modo, se x ∈ R, tal que x 6= 0, enta˜o
x · I =
{
(ax, bx) , se x > 0,
(bx, ax) , se x < 0.
De fato, se x > 0,
d ∈ (ax, bx)⇒ ax < d < bx⇒ a < d
x
< b⇒ d
x
∈ I
⇒ d = x · d
x
∈ x · I ⇒ (ax, bx) ⊂ x · I
e, reciprocamente,
d ∈ x · I ⇒ d = x · y, para algum y ∈ I ⇒ d
x
= y ∈ I
⇒ a < d
x
< b⇒ ax < d < bx⇒ d ∈ (ax, bx)⇒ x · I ⊂ (ax, bx) .
O caso a < 0 e´ ana´logo. Se B ⊂ R e´ aberto, enta˜o existe uma sequeˆncia In de intervalos
abertos dois a dois disjuntos tais que
B =
∞⋃
n=1
In.
Deste modo,
x+ B =
∞⋃
n=1
(x+ In)
x ·B =
∞⋃
n=1
(x · In)
34
e, como x + In e x · In sa˜o intervalos abertos para todo n ∈ N, segue que x + B e x · B sa˜o
conjuntos abertos. �
4. Sejam A e B abertos. Enta˜o os conjuntos
A+ B = {x+ y; x ∈ A, y ∈ B} e A ·B = {x · y; x ∈ A, y ∈ B}
sa˜o abertos.
SOLUC¸A˜O: Temos que
A+ B = {x+ y; x ∈ A, y ∈ B} = ⋃
x∈A
(x+ B) ,
A ·B = {x · y; x ∈ A, y ∈ B} = ⋃
x∈A
(x ·B) .
Do exerc´ıcio anterior segue que, para todo x ∈ A, x + B e x · B sa˜o conjuntos abertos e,
consequentemente, A+ B e A · B sa˜o abertos. �
5. Para quaisquer X, Y ⊂ R, tem-se
int (X ∩ Y ) = int (X) ∩ int (Y ) e int (X ∪ Y ) ⊃ int (X) ∪ int (Y ) .
Deˆ um exemplo em que a inclusa˜o na˜o se reduza a uma igualdade.
SOLUC¸A˜O: Temos que
a ∈ int (X ∩ Y )⇔ ∃ ε > 0; (a− ε, a+ ε) ⊂ X ∩ Y
⇔ ∃ ε > 0; (a− ε, a+ ε) ⊂ X e (a− ε, a+ ε) ⊂ Y
⇔ a ∈ int (X) e a ∈ int (Y )⇔ a ∈ int (X) ∩ int (Y ) .
Logo, todo elemento de int (X ∩ Y ) e´ tambe´m um elemento de int (X)∩ int (Y ) e vice-versa,
ou seja,
int (X ∩ Y ) = int (X) ∩ int (Y ) .
Do mesmo modo,
a ∈ int (X ∪ Y )⇒ ∃ ε > 0; (a− ε, a+ ε) ⊂ X ∪ Y
⇒ ∃ ε > 0; (a− ε, a+ ε) ⊂ X ou (a− ε, a+ ε) ⊂ Y
⇒ a ∈ int (X) ou a ∈ int (Y )⇒ a ∈ int (X) ∪ int (Y ) .
Logo, todo elemento de int (X) ∪ int (Y ) e´ tambe´m um elemento de int (X ∪ Y ), ou seja,
int (X ∪ Y ) ⊃ int (X) ∪ int (Y ) .
Fazendo X = [0, 1] e Y = [1, 2], temos que X ∪ Y = [0, 2]. Assim,
int (X ∪ Y ) = (0, 2) ) (0, 1) ∪ (1, 2) = int (X) ∪ int (Y ) .
�
35
6. Se A ⊂ R e´ aberto e a ∈ A enta˜o A− {a} e´ aberto
SOLUC¸A˜O I: Uma vez que A e´ aberto temos que Ac e´ fechado. Temos que
x ∈ (A− {a})c ⇔ x /∈ A− {a} ⇔ x ∈ Ac ou x ∈ {a} ⇔ x ∈ Ac ∪ {a} ,
ou seja, (A− {a})c = Ac ∪ {a}. Logo (A− {a})c e´ fechado por ser a unia˜o de conjuntos
fechados e, portanto, A− {a} e´ aberto. �
SOLUC¸A˜O II: Seja x ∈ A − {a}. Enta˜o x ∈ A e x 6= a. Uma vez que A e´ aberto segue
que existe δ > 0 tal que (x− δ, x+ δ) ⊂ A e, uma vez que x 6= a, |x− a| > 0. Seja
ε = min {δ, |x− a|}. Enta˜o a /∈ (x− ε, x+ ε) ⊂ A e, consequentemente, (x− ε, x+ ε) ⊂
A− {a}. Logo, todo ponto de A− {a} e´ interior e este conjunto e´, portanto, aberto �
SOLUC¸A˜O III: Seja I = (c, d) um intervalo aberto. Se a ∈ I, enta˜o I − {a} = (c, a) ∪ (a, d).
Com efeito,
x ∈ I − {a} ⇔ x ∈ I e x 6= a⇔ c < x < a < d ou c < a < x < d
⇔ x ∈ (c, a) ou x ∈ (a, d)⇔ x ∈ (c, a) ∪ (a, d) .
Se A e´ aberto enta˜o podemos escrever
A =
∞⋃
n=1
In,
onde os conjuntos In sa˜o intervalos abertos dois a dois disjuntos. Se a ∈ A, enta˜o existe
n0 ∈ N tal que a ∈ In0 e, consequentemente,
A− {a} =
(
n0−1⋃
n=1
In
)
∪ (In0 − {a}) ∪
(
∞⋃
n=n0+1
In
)
.
Logo A− {a} e´ uma reunia˜o de conjuntos abertos e e´, portanto, aberto. �
7. Considere as func¸o˜es f, g, h : R → R dadas por f (x) = ax + b (a 6= 0), g (x) = x2 e
h (x) = x3. Mostre que para cada A ⊂ R aberto, f−1(A), g−1 (A) e h−1 (A) sa˜o abertos.
SOLUC¸A˜O: Seja A ⊂ R um conjunto aberto. Enta˜o existe uma sequeˆncia (In) de intervalos
abertos dois a dois disjuntos tal que A =
∞⋃
n=1
In. Dada uma func¸a˜o ϕ : R → R temos que
ϕ−1 (A) = ϕ−1
(
∞⋃
n=1
In
)
=
∞⋃
n=1
ϕ−1 (In). Logo, e´ suficiente mostrar que se I = (c, d) e´ um
intervalo aberto, enta˜o f−1 (I), g−1 (I) e h−1 (I) sa˜o abertos.
• f (x) = ax+ b, a 6= 0: Supondo a > 0, temos que
x ∈ f−1 (I)⇔ f (x) ∈ I ⇔ c < ax+ b < d
⇔ c− b
a
< x <
d− b
a
⇔ x ∈
(
c− b
a
,
d− b
a
)
,
36
ou seja, f−1 (I) =
(
c− b
a
,
d− b
a
)
. Analogamente, verifica-se que se a < 0, enta˜o f−1 (I) =(
d− b
a
,
c− b
a
)
.
• g (x) = x2: Se c < d ≤ 0, enta˜o g−1 (I) = ∅. Se 0 ≤ c < d, enta˜o
x ∈ g−1 (I)⇔ g (x) ∈ I ⇔ c < x2 < d⇔ √c < |x| <
√
d
⇔ −
√
d < x < −√c ou √c < x <
√
d
⇔ x ∈
(
−
√
d,−√c
)
∪
(√
c,
√
d
)
,
ou seja g−1 (I) =
(
−√d,−√c
)
∪
(√
c,
√
d
)
. Se c < 0 < d, enta˜o (c, d) = (c, 0)∪{0}∪ (0, d).
Deste modo,
g−1 (I) = g−1 ((c, 0)) ∪ g−1 (0) ∪ g−1 (0, d)
= ∅ ∪ {0} ∪
(
−
√
d, 0
)
∪
(
0,
√
d
)
=
(
−
√
d,
√
d
)
.
• h (x) = x3: Temos que
x ∈ h−1 (I)⇔ h (x) ∈ I ⇔ c < x3 < d
⇔ 3√c < x < 3
√
d⇔ x ∈
(
3
√
c,
3
√
d
)
,
ou seja, h−1 (I) =
(
3
√
c, 3
√
d
)
. �
8. No exerc´ıcio anterior, mostre que, para cada A ⊂ R aberto, f (A) e h (A) sa˜o abertos.
Deˆ exemplo de A aberto tal que g (A) na˜o seja aberto.
SOLUC¸A˜O: Seja A ⊂ R um conjunto aberto. Enta˜o existe uma sequeˆncia (In) de intervalos
abertos dois a dois disjuntos tal que A =
∞⋃
n=1
In. Dada uma func¸a˜o ϕ : R → R temos que
ϕ (A) = ϕ
(
∞⋃
n=1
In
)
=
∞⋃
n=1
ϕ (In). Logo, e´ suficiente mostrar que se I = (c, d) e´ um intervalo
aberto, enta˜o f (I) e h (I) sa˜o abertos.
• f (x) = ax + b, a 6= 0: Supondo a > 0, temos que se y ∈ f (I) enta˜o existe x ∈ I tal
que y = ax+ b. Mas
x ∈ I ⇒ c < x < d⇒ ac < ax < ad
⇒ ac+ b < ax+ b = y < ad+ b⇒ y ∈ (ac+ b, ad+ b) ,
37
ou seja, f (I) ⊂ (ac+ b, ad+ b). Por outro lado,
y ∈ (ac+ b, ad+ b)⇒ ac+ b < y < ad+ b⇒ ac < y − b < ad
⇒ c < y − b
a
< d⇒ y − b
a
∈ I ⇒ y = f
(
y − b
a
)
∈ f (I) ,
ou seja (ac+ b, ad+ b) ⊂ f (I). Logo, f (I) = (ac+ b, ad+ b) e´ aberto. O caso em que a < 0
e´ ana´logo.
• h (x) = x3: Temos que se y ∈ h (I) enta˜o existe x ∈ I tal que y = x3. Mas
x ∈ I ⇒ c < x < d⇒ c3 < x3 = y < d3 ⇒ y ∈ (c3, d3) ,
ou seja, h (I) ⊂ (c3, d3). Por outro lado,
y ∈ (c3, d3)⇒ c3 < y < d3 ⇒ c < 3√y < d
⇒ 3√y ∈ I ⇒ y = f ( 3√y) ∈ f (I) ,
ou seja (c3, d3) ⊂ h (I). Logo, h (I) = (c3, d3) e´ aberto.
Seja agora A = (−1, 1). Enta˜o g (A) = [0, 1). Com efeito, se y ∈ g ((−1, 1)) enta˜o existe
x ∈ (−1, 1) tal que y = x2. Mas
x ∈ (−1, 1)⇒ 0 ≤ |x| < 1⇒ 0 ≤ |x|2 = x2 = y < 1⇒ y ∈ [0, 1) ,
ou seja g ((−1, 1)) ⊂ [0, 1). Por outro lado,
y ∈ [0, 1)⇒ 0 ≤ y < 1⇒ 0 ≤ √y < 1
⇒ √y ∈ [0, 1) ⊂ (−1, 1)⇒ y = g (√y) ∈ g ((−1, 1)) ,
ou seja, [0, 1) ⊂ g ((−1, 1)). Logo, g ((−1, 1)) = [0, 1) na˜o e´ aberto. �
9 Toda colec¸a˜o de abertos na˜o-vazios, dois a dois disjuntos e´ eumera´vel.
SOLUC¸A˜O: Seja {Aλ}λ∈Λ uma colec¸a˜o de conjuntos abertos dois a dois disjuntos. Do Teorema
da Estrutura dos Abertos da Reta segue que, para todo λ ∈ Λ, existe uma sequeˆncia (Iλn)
de intervalos abertos dois a dois disjuntos tal que
Aλ =
∞⋃
n=1
Iλn .
Deste modo, ⋃
λ∈Λ
Aλ =
⋃
λ∈Λ
(
∞⋃
n=1
Iλn .
)
Novamente, do Teorema da Estrutura dos Abertos da Reta segue que o conjunto de ı´ndices
Λ e´ enumera´vel, pois, caso contra´rio o aberto
⋃
λ∈Λ
Aλ seria escrito como uma unia˜o na˜o-
enumera´vel de intervalos abertos disjuntos dois a dois. �
38
10. O conjunto dos valores de adereˆncia de uma sequeˆncia e´ um conjunto fechado.
SOLUC¸A˜O: Seja (xn) uma sequeˆncia de nu´meros reais e seja X o conjunto dos seus valores de
adereˆncia. Se a ∈ X, enta˜o existe uma sequeˆncia (an) de elementos de X tal que a = lim an.
Do Exerc´ıcio 4.16 segue que existe uma subsequeˆncia de (xn) que converge para a, ou seja,
a e´ tambe´m um valor de adereˆncia de (xn). Logo X ⊂ X e o resultado segue. �
11. Se X ⊂ F e F e´ fechado enta˜o X ⊂ F .
SOLUC¸A˜O: Se a ∈ X enta˜o existe uma sequeˆncia (xn) de elementos de X tal que a = lim xn.
Como X ⊂ F , segue que a sequeˆncia (xn) e´ tambe´m uma sequeˆncia de elementos de F que
converge para a. Logo a ∈ F = F (pois F e´ fechado) e, portanto, X ⊂ F . �
12. Se lim xn = a e X = {x1, x2, . . . , xn, . . .} enta˜o X = X ∪ {a}.
SOLUC¸A˜O: Se X e´ um conjunto finito enta˜o e´ fechado. Se X e´ infinito enta˜o X ′ = {a}.
Com efeito, se existisse b ∈ X ′ tal que b 6= a, enta˜o tomando ε1 > 0 existiria n1 ∈ N
tal que xn1 ∈ (b− ε1, b+ ε1) e xn1 6= b. Tomando ε2 =
|xn1 − b|
2
, existiria n2 ∈ N tal
que xn2 ∈ (b− ε2, b+ ε2) e xn2 6= b. Tomando ε3 =
|xn2 − b|
22
existiria n3 ∈ N tal que
xn3 ∈ (b− ε3, b+ ε3) e xn3 6= b. Procedendo desta forma, obtemos uma subsequeˆncia (xnk)
de (xn) que converge para b contradizendo o fato de ser lim xn = a. Logo, como X = X ∪X ′
temos enta˜o que X = X ∪ {a}. �
13. O nu´mero
1
4
pertence ao Conjunto de Cantor
SOLUC¸A˜O: Seja
[
a
3k
,
a+ 1
3k
]
um intervalo remanescente apo´s a retirada dos terc¸os me´dios na
k-e´sima etapa de contruc¸a˜o do conjunto de Cantor. Temos que[
a
3k
,
a+ 1
3k
]
=
[
a
3k
,
a
3k
+
1
3k+1
]
∪
[
a
3k
+
1
3k+1
,
a+ 1
3k
− 1
3k+1
]
∪
[
a+ 1
3k
− 1
3k+1
,
a+ 1
3k
]
Logo, na etapa seguinte de construc¸a˜o do conjunto de Cantor teremos que os intervalos
remanescentes apo´s a retirada do terc¸o me´dio relativo ao intervalo
[
a+ 1
3k
− 1
3k+1
]
sera˜o os
intervalos [
a
3k
,
a
3k
+
1
3k+1
]
e
[
a+ 1
3k
− 1
3k+1
,
a+ 1
3k
]
.
Provamos enta˜o que se
p
3k
e
q
3k
sa˜o extremos superior e inferior, respectivamente, de algum
intervalo remanscente apo´s a k-e´sima etapa de contruc¸a˜o do conjunto de Cantor, enta˜o
os valores
p
3k
− 1
3k+1
e
q
3k
+
1
3k+1
sera˜o, respectivamente, extremos inferior e superior de
intervalos remanescentes apo´s as retiradas dos trec¸os me´dios na etapa seguinte da construc¸a˜o
39
do conjunto de Cantor. Uma vez que
1
3
e´ extremo superior de um intervalo remanescente na
primeira etapa de construc¸a˜o do conjunto de Cantor temos que a sequeˆncia (xn) dada por
x1 =
1
3
x2 =
1
3
− 1
32
x2 =
1
3
− 1
32
+
1
33
...
xn =
n∑
k=1
(−1)k+1
3k
...
e´ uma sequeˆncia de extremos do conjunto de Cantor. Temos que
lim xn =
∞∑
n=1
(−1)n+1
3n
=
1
3
∞∑
n=0
(−1)n
3n
=
1
3
· 1
1−
(
−1
3
) = 1
4
.
Do fato de o conjunto de Cantor ser fechado segue o resultado. �
14. Sejam F , G conjuntos fechados disjuntos tais que F ∪ G seja um intervalo fechado
(limitado ou na˜o). Enta˜o F = ∅ ou G = ∅.
SOLUC¸A˜O:
�
15. Seja E ⊂ R enumera´vel. Consiga uma sequeˆncia cujo conjunto dos valores de adereˆncia
e´ E. Use este fato para mostrar que todo conjunto fechado F ⊂ R e´ o conjunto dos valores
de adereˆncia de alguma sequeˆncia.
[Sugesta˜o: Escreva N como reunia˜o enumera´vel de conjuntos infinitos disjuntos Ni. para
cada n ∈ Ni fac¸a xn = i-e´simo elemento do conjunto E. Para a segunda parte use o Teorema
6.]
SOLUC¸A˜O:
�
40

Outros materiais

Materiais relacionados

Perguntas relacionadas

Perguntas Recentes